Posts for p4wn3r

1 2
25 26 27
34 35
Experienced Forum User, Published Author, Player (42)
Joined: 12/27/2008
Posts: 873
Location: Germany
Suggestion: Use a change of variables that rotates the axes by an angle of 45º Bonus points if you can see the mathematical importance of the result of this integral.
Post subject: Re: The 43 Crocodile Army
Experienced Forum User, Published Author, Player (42)
Joined: 12/27/2008
Posts: 873
Location: Germany
RingRush wrote:
1. Get a level 0 bad clone (involves the classic cloning glitch, resetting at the exact right time...there is more on this on plenty of sites).
Do you have a vbm getting this bad clone? I tried many times, but couldn't get a lvl0 bad clone on VBA-rr no matter which frame I reset.
Experienced Forum User, Published Author, Player (42)
Joined: 12/27/2008
Posts: 873
Location: Germany
Four years ago...
primorial#soup wrote:
Chamale wrote:
BRILLIANT IDEA! Start with Squirtle instead of Charmander. In Viridian Forest, run into a wild Pikachu in front of a trainer (I've had this happen on console). Get owned by Pikachu. You white out and return to the last pokemon centre, thus triggering a trainer-fly glitch.
Hmm. I experimented earlier with this, fainting from poison directly in front of a trainer (result, teleported directly without triggering the glitch). I'll experiment with this as well.
It seems someone *DIDN'T* experiment...
Experienced Forum User, Published Author, Player (42)
Joined: 12/27/2008
Posts: 873
Location: Germany
Well, it's been here long enough. The method to solve: Because of the small concavity, any force that will act during the collision will be applied at the new axis of rotation. Thus, there won't be any torque with relation to this axis and angular momentum will be conserved. From this relation, we can get the ratio of the angular velocity of the prism before the impact to the one after it. Since there's no sliding, the prism only rotates and its kinetic energy is only rotational and proportional to the square of its angular velocity. From this, we get the ratio of energy lost during the collision. We're also able to deduce the relation of the kinetic energy of the pencil immediately before the (n+1)th collision to the energy immediately before the nth collision. It can be seen as the amount of energy that didn't dissipate in the collision plus the potential energy it gained while going down the plane. We find the limit of the kinetic energy as n goes to infinity. If this limit is too small, the prism will eventually hit the plane, lose some energy and won't have enough mechanical energy to lift its center of mass to the highest point and won't be able to continue its movement. Therefore, the smallest angle is the one that gives just enough asymptotic kinetic energy to lift the pencil after a collision. This final equation will give the angle. The execution is not as pretty. First of all, the angular momentum of a rigid body is intimately related to its moment of inertia, so we have to calculate it for the pencil. To do this, instead of considering an hexagon with a central axis, we calculate the moment of inertia for a regular triangular prism rotating along its edge and multiply the result by six. So, considering the edge of the hexagonal of length a, its density rho and its longitudinal length L, the moment of inertia is given by the double integral: It's convenient to express it in function of the mass, so by using the formula for the volume of an hexagonal prism: Using Steiner's theorem, we find the moment for the rotation about one of the pencil's edges: Now we're ready to analyze the motion. The following pictures show the pencil before, during and after a collision, respectively: http://i992.photobucket.com/albums/af49/p4wn3r/hexagonbefore.png http://i992.photobucket.com/albums/af49/p4wn3r/hexagonduring.png http://i992.photobucket.com/albums/af49/p4wn3r/hexagonafter.png Notice that point B is at rest before the collision, so it suffers no force from the plane. The small concavity implies that all interaction will occur at point A only, and since collisions take short time, force Fc is much larger than the weight, normal and static friction. In a theoretical situation where they take infinitesimal time, the effect of those other forces is zero and we can assume that only Fc is acting during the collisions. Thus, we can find a relation between the two rotating movements noticing that angular momentum is conserved with respect to A. So, we calculate the angular momentum before the collision. It can be seen as the addition of the angular momentum about the center of mass plus the momentum the prism would cause in A if its mass were all shrunk in its center of mass: The angular momentum after is easier to find, because it's only a rotation about A: From the equality of these formulas, we get that the ratio of angular velocities after the collision is 11/17. Since K = I omega^2 / 2, we get the amount of energy that dissipated: Let K_n denote the energy immediately before the nth collision. Before the (n+1)th collision, it'll have 121/289 of the energy it had before plus the potential energy it gained from the descent of the center of mass. Geometrically, after a 60º revolution, the center of the hexagon is a distance of "a" units from where it was before, on a line inclined \theta from the horizontal. So, the vertical descent of it is a*sin(theta) and: It can be proved that, for small enough angles, this sequence converges (you can solve the linear recurrence or prove it's bounded and monotonic after a certain point). Say that the limit is K, then: Finally, we notice that, for the movement to continue, K cannot be too small. At the smallest angle, the kinetic energy the prism has after hitting the plane is just enough to give it enough potential energy to lift its center of mass to the highest position in the trajectory. Geometrically, we can see that the segment OA is (30º-\theta) from the vertical. When it rotates, the difference of height of the center will be a(1-cos(\theta)). By equaling the remaining kinetic energy after collision with the potential energy to lift it this high, we get the final equation (I use u=121/168): This last equation can be solved as any other type of a cos(x) + b sin (x) = c . We divide it by sqrt(a^2+b^2) and now the coefficients multiplying sin and cos obey the fundamental trigonometric relation and are the sine and cosine of another angle, and we can simplify it applying the sin(a+b) formula. I know many other physics problems whose solutions will make this one look very small :P
Experienced Forum User, Published Author, Player (42)
Joined: 12/27/2008
Posts: 873
Location: Germany
Derakon wrote:
Ah, then you're talking about a non-ideal situation, and it's generally a good idea to clarify that. :) You did mention friction preventing sliding, but it's usually a good idea to specify that some energy will be lost from these interactions.
Okay, interpretation is often a part of the question, but I'll clarify it. The situation is really non-ideal, there's friction and the prism can lose energy interacting with the plane, more specifically when it collides with it. Other than that, you don't need to consider other friction forces in the rolling motion besides the static one. As many have said, the actual friction coefficient doesn't matter, it's just necessary to keep in mind that it prevents sliding and friction/other dissipative forces can take some of its energy in the collision. In your latest post, I think you got the idea.
marzojr wrote:
Hrm. That problem is more annoying than difficult.
If you say so... I think it depends on what one thinks where difficulty lies, on the complexity and abstract nature of the more advanced concepts or in the continuous application of theorems/laws of the subject. Personally, I think the latter poses greater difficulty because providing shorter solutions to potentially exhaustive problems takes a lot more experience to master. However, I have to admit that physics is not very flexible to problem making. While in math, for example, you can make a problem solvable in 10 lines that will pose a challenge to many research mathematicians, in physics this is considerable harder, so difficulty is often achieved by making the problem bigger. I understand that many users won't have the time nor the interest to do all the necessary algebra though. I'll consider the problem solved if someone gives a precise description of the method and laws/theorems used to achieve the solution.
Experienced Forum User, Published Author, Player (42)
Joined: 12/27/2008
Posts: 873
Location: Germany
Derakon wrote:
There must be some energy loss for the pencil to stop. In an ideal setting (no losses from friction, air resistance, etc. and no outside inputs after the initial impulse), I'm not seeing where this energy would go. Thus in such a setting, the pencil should continue rolling indefinitely even on a level surface. As soon as you slope the surface up, though, the pencil will start making net KE -> PE trades with each revolution, and will eventually lack the KE to push itself over the energy barrier to the next revolution. My question then is, what is the uphill slope at which the pencil will start rolling back down instead of coming to a stop? :)
There's friction in this problem, it clearly mentions "friction prevents any sliding". Also, non-elastic collisions reduce the mechanical energy of the prism. This problem was actually proposed at an important physics event, there's nothing wrong with it. And yes, by horizontal axis, it means that the prism is aligned with the plane.
Experienced Forum User, Published Author, Player (42)
Joined: 12/27/2008
Posts: 873
Location: Germany
I like classical mechanics more. I'll post a problem I could solve with some considerable amount of work. I think people here will find it interesting. Alice is a student that doesn't like paying attention to her classes. Instead, she likes to do physics experiments while her teachers are speaking. In one occasion, she placed one of her pencils (which had the form of a regular hexagonal prism) in the top of her table, an inclined plane, and started giving minor impulses to the pencil so that it rolled down the table. Eventually, she realized that when her initial impulse was enough to make it roll once, the pencil always rolled down non-stop to the end of the table. She concluded that this happened because her table was inclined by a significantly high angle and if the table's inclination were lighter, the pencil could stop before reaching the end. Since she wasn't paying attention to mechanics class, she asks you to analyze this phenomenon for her. Consider a long, solid, rigid, regular hexagonal prism with uniform mass distribution. The prism is initially at rest with its axis horizontal on an inclined plane which makes a small angle \theta with the horizontal. Assume that the surfaces of the prism are slightly concave so that it only touches the plane at its edges. The effect of this concavity on its inertial properties can be ignored. The prism is now displaced from rest and starts an uneven rolling down the plane. Assume that friction prevents any sliding and that the prism does not lose contact with the plane. Calculate the smallest angle \theta, for which the uneven rolling, once started, will continue indefinitely.
Experienced Forum User, Published Author, Player (42)
Joined: 12/27/2008
Posts: 873
Location: Germany
OK, now let me try to understand what you did. I read some things on the internet, but most of marzojr's solution I ended up deducing myself, I have little knowledge of special relativity besides high school introduction. See if my considerations are correct. First of all, you define the motion of the particle in a 4-D vector space containing time and its x,y,z axis positions. Then, instead of using the Euclidean norm, you introduce some form of Riemannian-like metric (which is technically not a metric, because it can be negative sometimes, but I think this fact shouldn't be a problem here) that is invariant of the coordinate system provided you take a base that's orthonormal with regard to this "inner product". By taking note of this fact, if you're analyzing movement from a certain frame, you can take the norm of the time-position vector (I'll call it X), defined as the square root of the Minkowski product of the vector with itself. The postulates of relativity imply that this product is always positive and the norm is the time that passed for the mover, called proper time. Thus, its well defined for every coordinate system. Because differentiation and base changes are linear operators and more specifically, the operator changing from and to orthonormal bases is a self-adjoint operator, one can differentiate X as many times as he wants and all d^nX/dT^n vectors will have the same norm for any coordinate system. This is specially useful for velocity and acceleration with respect to T. Because the norm (or maybe the square of the norm?) will always be the same, one can compare it with a trivial case to find its value. The most simple is a frame that's moving together with the particle. To find the norm of proper velocity, you simply look at the vector (c,0,0,0) which has norm c^2. For the norm of proper acceleration, it's trickier because you have to differentiate more, but for the frame of the ship, one can find -a^2. Thus, on Earth's frame, if you take the 4-D velocity and acceleration vectors and use the results that their norms must equal c^2 and -a^2, you arrive at a system that can be transformed into one of non-linear first order ODE's, which can be solved by undetermined coefficients. Also, I believe this is a typo, because it should have a square root, but didn't influence the posterior steps:
marzojr wrote:
Ut(τ) = dt(τ)/dτ = 1 / {1 - v(τ)2 / c2}
Experienced Forum User, Published Author, Player (42)
Joined: 12/27/2008
Posts: 873
Location: Germany
marzojr wrote:
We must calculate the final value of τ we are interested in; this corresponds to the half-way point x(τm) = (4.365/2) ly. To do this, lets invert the equation for x(τ) -- we can do that in this interval because x(τ) increases monotonically with τ in this interval -- to obtain τ = (c / a) * acosh{(a / c2) * x(τ) + 1} Plugging in x(τm) = (4.365/2) ly = 2.1825 ly, a = 1.0315512 ly/y2 and c = 1 ly/y, we find τm = 1.791172306 y For the Earth reference frame, this corresponds to t(τm) = 2.999132811 y Multiplying these by 4 for the four segments of the journey, we see that 7.164689224 y transpire at the rocket, while 11.996531244 transpire on Earth. The difference to p4wn3r's result is probably a result of numerical errors in his part, as his math seems correct.
Thanks for looking over my "solution". I didn't do the results very precisely, but still the time for the rocket is differing by a large value. Now I have a question, why can you you plug half the distance from the star directly into that formula? Shouldn't the distance for the ship's frame shrink because of the Lorenz factor? EDIT: Never mind, I've read it properly now, I evaluated my results in a calculator and got the same thing, it's not easy to evaluate an ln by hand...
Experienced Forum User, Published Author, Player (42)
Joined: 12/27/2008
Posts: 873
Location: Germany
I just finished that problem. I'm no physicist, all I recall is that special relativity is like classic mechanics with some extra square roots in ODEs. A physicist would probably get the solution better, I only know how to solve ODEs. To model constant acceleration, I considered the quotient of the force to the invariant mass to be constant. I did everything by hand and I'm not sure if it's correct. I don't have time to fully comment now so I'll just post the calculations Warp wants to see (some were omitted, but it's just boring integration/differentiation/algebraic manipulation that I didn't need to write to do). D is half the distance from Earth to Alpha Centauri in Earth's static frame, T is the time it takes to get to 1/4 of the trip measured in the Earth and T' the time measured in the ship, v is the velocity of the ship and x its position from earth's view. After you pass the D distance, the situation is symmetric (the integrals are the same, just with some changes of variables), so it suffices to do the problem for first 1/4. Someone please tell me if I did anything wrong. (I did (2) first) For this problem, k = 2,24, and I found the times: (1): 4T' = 6,20 years (2): 4T = 11,95 years At least for me, this looks coherent :P
Experienced Forum User, Published Author, Player (42)
Joined: 12/27/2008
Posts: 873
Location: Germany
It suffices to have a = 4c^4 for c>=2.
Nitrodon wrote:
(n^2+2cn-2c^2)(n^2-2cn-2c^2) = n^4 + 4c^4
Wrong, Sophie Germain's identity says: n^4 + 4c^4 = (n^2 - 2nc + 2c^2)(n^2 + 2nc + 2c^2) Each factor is (n +/- c)^2 + c^2 >= c^2 >= 4.
Experienced Forum User, Published Author, Player (42)
Joined: 12/27/2008
Posts: 873
Location: Germany
(IMO 1969) Prove that there are infinitely many positive integers a such that the sequence (zn)n>=1, zn=n4+a does not contain any prime number.
Experienced Forum User, Published Author, Player (42)
Joined: 12/27/2008
Posts: 873
Location: Germany
Do you mean the remainder in polynomial long division of smallest degree? 15x4 - y2 = (x2+y) (15x2 - 15y) + 14y2
Experienced Forum User, Published Author, Player (42)
Joined: 12/27/2008
Posts: 873
Location: Germany
Nice solution, FractalFusion. Here's one using elementary geometry: Take points F and G, the intersection of lines AE and BE with the smaller circle, respectively. Since the angle FEG = 90º, the triangle FEG is right and is inscribed in a semicircle. Therefore, FG is the diameter of the smaller circle and the points F, C and G are collinear. Now, take O as the center of the big circle, and draw the line OE. Since the circles are tangent, this line passes through C. Since the triangle AOE is isosceles, angle OAE = angle OEA = angle CEF. But, since the triangle ECF is isosceles, angle CEF = angle EFC = angle EFG. Finally, the triangles ABE and FGE have two equal angles and are similar. Knowing this similarity, we finally bring D into the story. By power of a point, AD^2 = AF AE and BD^2 = BG BE . Dividing the equations: (AD / BD)^2 = (AE / BE) * (AF / BG) . Using the similarity, (AD / BD)^2 = (AE / BE)^2 , since the ratios are positive, AD/BD = AE/BE. The last conclusion allows us to use the angle bisector theorem. Therefore, ED bisects the right angle AEB, so AED = 45º Q.E.D
Experienced Forum User, Published Author, Player (42)
Joined: 12/27/2008
Posts: 873
Location: Germany
Olá wwmarx. Não conheço nada de Final Burn Alpha, então traduzi sua pergunta e enviei por PM ao DarkKobold, que tem bastante experiência com ele.
DarkKobold wrote:
Unfortunately, FBA is just a crappy emulator, and none of the current coders on the site are actively maintaining it. Game-based crashes are not going to be fixed any time soon.
Então, ao que parece, o emulador tem sérios problemas e não há nenhum programador aqui trabalhando nele ativamente, portanto não há previsão para consertá-lo. Sinto muito :(
Experienced Forum User, Published Author, Player (42)
Joined: 12/27/2008
Posts: 873
Location: Germany
There's more than one way to inscribe the smaller circle, it's not required that AD=BD, you must prove that the angle AED is still 45º for all possible ways to inscribe the smaller circle. EDIT: A picture is worth over nine thousand words:
Experienced Forum User, Published Author, Player (42)
Joined: 12/27/2008
Posts: 873
Location: Germany
Am I starting to get annoying with roughly one problem per weekend? I'll stop if someone tells me to. Anyway... The segment AB is the diameter of a semicircle. A smaller circle of center C is inscribed in this semicircle. Let D be the tangency point of the circle with the diameter AB and E the tangency point of the circle with the arc AB. Find the measure of the angle AED.
Experienced Forum User, Published Author, Player (42)
Joined: 12/27/2008
Posts: 873
Location: Germany
Nach wrote:
Can each of you who is expert in your language write how to say "<language> forum" in your language? I'm going to use it for a link on the site. Thanks.
Portuguese: "Fórum em português"
Experienced Forum User, Published Author, Player (42)
Joined: 12/27/2008
Posts: 873
Location: Germany
The Feynman Lectures on Physics, by Richard Feynman -- Physics (duh) The Lady and the Tiger, by Raymond Smullyan -- Logic Introduction to Algorithms, by Cormen, Leiserson, Rivest and Stein -- Computer Science
Experienced Forum User, Published Author, Player (42)
Joined: 12/27/2008
Posts: 873
Location: Germany
I posted a message in the Portuguese forum saying how it should be used. I tried to remain as faithful as possible to Nach's message. The non English forums are a great idea, I think they'll be used often.
Post subject: Boas vindas ao sub-fórum de língua portuguesa
Experienced Forum User, Published Author, Player (42)
Joined: 12/27/2008
Posts: 873
Location: Germany
Este sub-fórum foi criado com a intenção de auxiliar a comunicação entre os autores de TAS que têm o português como língua nativa, visto que alguns tem certa dificuldade com o idioma principal do fórum, o inglês. Aqui devem ser discutidos quaisquer aspectos relacionados a TAS sem a necessidade de traduzir para o idioma oficial do site. Os usuários novos que falam português são encorajados a postar questões relacionadas ao entendimento de TASes em geral, o processo de submissões, regras e apresentação de trabalhos em progresso. Também podem ser pedidos esclarecimentos sobre termos típicos do jargão de TAS e traduções de material relacionado ao assunto encontrado em outras línguas. A discussão sobre os sub-fóruns de idiomas estrangeiros em geral e seu uso é feita neste tópico: http://tasvideos.org/forum/viewtopic.php?t=11067 Esperamos que este sub-fórum ajude na integração do site TASVideos à comunidade de língua portuguesa. Divirtam-se. ;)
Experienced Forum User, Published Author, Player (42)
Joined: 12/27/2008
Posts: 873
Location: Germany
I already knew "it" was the correct pronoun to use. I used "her" mostly as a tiny joke, in a sense close to what Warp pointed out, like pets or ships. Portuguese translation should be "Agora termine-a". In Portuguese pronouns are different when used as direct or indirect objects. "lhe/lhes" are for indirect objects and "o/a/os/as" for direct objects.
Experienced Forum User, Published Author, Player (42)
Joined: 12/27/2008
Posts: 873
Location: Germany
I wonder how many flame wars certain N64 games have created here (and how many they will start in the future).
Experienced Forum User, Published Author, Player (42)
Joined: 12/27/2008
Posts: 873
Location: Germany
I actually found it fun to put x back into the formula. It turned out much prettier than I thought it would. A fast way to get x is: It's a good exercise to simplify the expression and put x in it, we eventually get: Eat that Mathematica! To make sure it was correct, I plugged it in Mathematica itself and...
OmnipotentEntity wrote:
That doesn't looks like an easy integration to do. Why do you think that Wolfram Alpha is wrong and there's a trivial anti-derivative to that function?
Because there's a criterion that I can apply to that function that will say it has an elementary primitive shortly after looking at it, it's an obscure technique that I didn't offer a solution that used it to show it wasn't necessary, but I'll tell it now. Look at the change of variable that ultimately solved the integral, namely v in terms of x. To find it through normal means, we had to apply two changes of variables that required some experience with integrals and some algebra. The question is: is there a method that would determine right off the bat the transform that would make the integral solvable by the common methods (integration by parts, partial fractions, etc) ? The answer is yes, for that function it's relatively easy. This is called Chebyshev's finite integrability theorem. Functions of the form f(x) = x^m ( a + b x^n )^p , where m,n,p are rational numbers, admit an elementary primitive if, and only if at least one of the following is satisfied: 1) p is an integer. For this case, expand the binomial and integrate the function. 2) (m+1)/n is an integer. For this case, apply the change of variable a + b x^n = u^s , where s is the denominator of the irreducible fraction of p. 3) (m+1)/n + p is an integer. For this case, apply the change of variable a x^(-n) + b = u^s, where s is the same as in (2). If you look at the function, it satisfies case (3), so it must have an elementary primitive, and you could get it done much sooner with this, but I wanted to show it wasn't necessary. I'm surprised this simple procedure was missed by the integrator's programmers.
Experienced Forum User, Published Author, Player (42)
Joined: 12/27/2008
Posts: 873
Location: Germany
It's right. You need to do another change of variable now: The more straight forward would work too, and it's how I first integrated the function, while rearranging the resulting terms, I noticed the previous one would get the job done sooner. Now finish her.
1 2
25 26 27
34 35